LSAT and Law School Admissions Forum

Get expert LSAT preparation and law school admissions advice from PowerScore Test Preparation.

User avatar
 Dave Killoran
PowerScore Staff
  • PowerScore Staff
  • Posts: 5850
  • Joined: Mar 25, 2011
|
#26372
Complete Question Explanation
(The complete setup for this game can be found here: lsat/viewtopic.php?t=6413)

The correct answer choice is (E)


If Q is given in the morning, the only morning that can occur is Wednesday morning. Additionally, from the first rule J must be given with K. From the second rule, R must be given with N or S, and that block must be placed on Thursday or Friday. This presents a problem for Wednesday afternoon: only Q, R, or S can be given on Wednesday afternoon, but Q is given Wednesday morning, and R is given Thursday or Friday afternoon. Thus, only S can be given on Wednesday afternoon, leaving N to be given on the same day as R. This information results in the following setup:
powerscore_M12_T3_J2010_LG_explanations_game_1_#5_diagram_1.png
Accordingly, S cannot be given on Thursday, and answer choice (E) cannot be true and is correct.
 deck1134
  • Posts: 160
  • Joined: Jun 11, 2018
|
#49295
Why is it not possible to have N/S as a block instead of RN, and then get a completely different answer? That is what happened to me on this question.
 deck1134
  • Posts: 160
  • Joined: Jun 11, 2018
|
#49296
Nevermind! I forgot the restriction on R by rule #2.

Get the most out of your LSAT Prep Plus subscription.

Analyze and track your performance with our Testing and Analytics Package.